Social Question

Rarebear's avatar

Do you think there is a "mind" or is there just "brain"?

Asked by Rarebear (25192points) June 25th, 2011
205 responses
“Great Question” (11points)

As a pure empiricist, I believe the “mind” is a made up construct by our species. I think our thought processes are merely biochemical and bioelectrical pathways in the brain and if that’s damaged then it ceases to function. But many people here will disagree with me that we do have a “mind”, and some will even say that this mind is transcendent. Hogwash I say! What say you?

(And don’t even get me STARTED on the word “soul”!)

Observing members: 0
Composing members: 0

Answers

GracieT's avatar

I may be missing the point entirely, but exactly why is it that even the mention that a brain is damaged means that there is no longer any mind?

Rarebear's avatar

@GracieT I’m saying the mind does not exist. Our thought processes are merely biochemical and electrical reactions in our brain. Let me try to reword it.

There I reworded it. Does it make more sense now?

WasCy's avatar

It’s a good question, and definitely worth the exploration.

It seems, though, that many times we’ve considered people to be “brain damaged” it’s really the interfaces that are damaged. I’m thinking, for example, about locked-in syndrome which is one of my ultimate horrors.

Edit: (I didn’t finish the thought.)

So maybe there are other, similar occurrences where “we believe” the brain to be damaged, or maybe it really is damaged, but the person’s mind (wherever that resides, or however it’s accessed) is available for the person himself, but no communication is possible because of the damage to the brain / interfaces.

Rarebear's avatar

@WasCy I actually have a patient with locked-in syndrome. Fortunately we diagnosed it and we treat him appropriately. Some brain damaged people still have function, certainly.

jaytkay's avatar

why is it that even the mention that a brain is damaged means that there is no longer any mind?

I think one could say there is not the same mind.

JLeslie's avatar

I use the words differently. Brain is the organ, while mind is the outward expression of the brain I guess? In the end I guess we, our thoughts, personality, knowledge is simply our brains, the wiring, neurons, filing systems, that make us what we are. For me, what I think os important, is to believe it is not hard wired. I have had my own experience in rewiring my brain, or that is what I call it. Changing my thoughts, reframing events, which has changed my life, my mind, even my physical response to things.

When there is brain damage, depending how severe, the mind is changed or damaged. The person is not the same person, if that is what you mean by transcendent. In fact, even without brain damage, but just with a person going through a significant change in their life, a life altering experience, or a big self discovery, the mind, and the person might be very different. The whole brain might be intact, with the same neural pathway capabilities, but the brain has been rewired for different areas to respond.

GracieT's avatar

@Rarebear, thank you for clarifying that. I have to agree with @WasCy: being victim of anything such as locked-in syndrome is terrifying to me! I know that now I have a somewhat minor problem with not being able to adequately express my thoughts. That is terrifying enough.

lillycoyote's avatar

I think they are just words but they also describe, as best we can with language, two separate things. The mind is a creation of the brain, it is a bio-electro-chemical physically based thing that we, I think are far from understanding completely. But the fact that something has biological basis doesn’t reduce it’s importance, doesn’t make it any less real or amazing.

That we are capable of consciousness, of self-awareness, of thought, creativity, intuitive understanding, all those mind things, if it has a biological basis, it still means that there such a thing a mind that exists and I don’t think it can be said to have an existence that is separate or outside, exactly, just that when comes to the human mind and the human brain, the brain is “more than the sum of it’s parts”, it’s biological processes if that makes any sense.

And that what is essential a physiological, biological machine can do something as extraordinary as create what with think of as the “human mind” that just so amazing it’s mind-boggling sometimes, or brain-boggling, depending on the position you take

Rarebear's avatar

@jaytkay Well, I change my mind all the time :-).

My point is that from a purely empirical and scientific point of view, there is just a brain—either working or damaged, or whatever. The word “mind” is a purely philosophical construct.

jaytkay's avatar

Locked-in syndrome…complete paralysis of nearly all voluntary muscles in the body except for the eyes.

Really? It’s always the eyes? The eyes are universally the last to go?

What if it were your toes and nobody noticed while they searched your eyes for activity?

Rarebear's avatar

@jaytkay My patient can only follow me with his eyes. Although he’s moving his head very slightly now.

flutherother's avatar

An analogy that comes to mind is the face. You could say it is the front of the head and consists of nothing more than two eyes a nose and a mouth. That is all a face actually is but of course a face is a lot more than the sum of its parts. A face is a construct of our species but then so is most of reality.

Ladymia69's avatar

What of situations where they brain is supposed to be dead, but comes back?

Mariah's avatar

I’m inclined to believe that same, @Rarebear, but I do still think consciousness is a great mystery.

Ladymia69's avatar

@Rarebear How can you be sure in your own head of anything that has to do with mind? Do you get your assurance because you may have seen the brain itself in your dealings as a doctor, and it just looks like a fallible, simple piece of meat to you?

Just wondering how you decided that it was only electrical impulses in water and flesh.

MRSHINYSHOES's avatar

I think the mind, our thought processes, are controlled and operated by the brain, and when the brain is gone, we cease to have a mind. In that respect, we cease in the physical sense, but we continue with a soul until we are re-born into the next life, when we assume a physical body again, and hence a brain and accompanying mind.

jerv's avatar

A computer is just silicon and copper, and it actually doesn’t do much of anything.

Software allows you to access a global information archive, watch and edit movies and music, and far more.

They are interdependent, but separate. You cannot run software without a computer, but a computer without software is useless. By the same token, a brain without a mind is merely a squishy, inert mass. As for whether we can have a mind without a brain (like ghosts), I leave that one to the philosophers and fiction authors.

ninjacolin's avatar

For me, the mind is the output of a functioning brain. Kinda like how a functioning projector and reel don’t make a movie but the images on the big screen, the product, is the movie.
If the projector or film is damaged, you get a shitty movie.

Coloma's avatar

I’m the open minded psycho/spiritual mystic here, I think that regardless of science there are unsolved mysteries and to be truly open ‘minded’ one must be just that.

Bottom line, we know no-thing for certain.

There was a case of a guy in a coma for something like 20 years not long ago that turned out, through using some sort of computer program to be able to communicate that he was, infact ‘conscious’, aware and had simply been trapped in a dead zone of paralization but with full awareness for years. I cannot remember the source, but it is true.

Good thing nobody pulled the plug because his brain waves seemed feeble.

I also just saw a documentary where a study was done that said that on a collective basis most doctors will not admit to any ‘spiritual’ beliefs, will attribute certain healings as spontaneous healings, BUT..if questioned on an individual basis many said they did believe in a higher power.

At the very least they had a willingness to set ego aside and simply say those 3 little words so many have such trouble with ” I don’t know.”

InTheZone's avatar

If the brain is all that there is, it seems to me that identical twins would always be virtually identical in personality and function. In fact, we know that in many instances they can be vastly different in personality and in the way their minds work. I understand that small differences in pre-natal influences could account for some variations, but I wouldn’t think that the extent of differences would be quite so dramatic as they are in some sets of identical twins.

I think that the mind is what we do with the brain we are born with. As @jerv said, I see it as similar to software operating the computer. Software is as real as the hardware it operates.

Mariah's avatar

@InTheZone Saying that the brain is a purely physical object does not imply that it cannot be altered by life experiences.

Isn’t software a non-physical phenomenon resulting from the interactions of physical objects (hardware)? I’m not completely understanding the analogy.

Rarebear's avatar

@JLeslie “The whole brain might be intact, with the same neural pathway capabilities, but the brain has been rewired for different areas to respond.”

That’s exactly my point. It’s simply the wiring of the brain that determines our consciousness and personality.

@flutherother “That is all a face actually is but of course a face is a lot more than the sum of its parts. A face is a construct of our species but then so is most of reality.”

Have you heard of face blindness? It’s a neurologic condition where people cannot recognize facial differences. And I would argue that the only reason why we recognize face is that we’re evolutionarily selected for that. You absolutely need to tell friend from foe, and very quickly.

@ladymia69 “What of situations where they brain is supposed to be dead, but comes back?”

Key supposition in your question is “supposed to be”.

@ladymia69 “Just wondering how you decided that it was only electrical impulses in water and flesh.”

I didn’t decide this. It’s what I believe the evidence shows. And I didn’t say only electrical impulses, I said biochemical and bioelectrical impulses.

@mrshinyshoes “I think the mind, our thought processes, are controlled and operated by the brain, and when the brain is gone, we cease to have a mind.”

That’s a circular statement. The mind is caused by the brain, and if the brain doesn’t exist we don’t have a mind.

@jerv You’re saying you think the mind is software?

@mariah “but I do still think consciousness is a great mystery.”

Sure. But just because it’s a mystery does not mean that there is not a purely empric scientific explanation for it.

@coloma “I also just saw a documentary where a study was done that said that on an individual basis most doctors will not admit to any ‘spiritual’ beliefs, will attribute certain healings as spontaneous healings, BUT..if questioned on an individual basis many said they did believe in a higher power.”

But I’m not really talking about spiritualism here. That’s a subject for another thread. And I’m not even saying that my empirical view would be held by most doctors. I’m just saying that I see no evidence of a mind separate from a brain. So to me, the mind does not exist and there is just the brain.

@inthzone “If the brain is all that there is, it seems to me that identical twins would always be virtually identical in personality and function.”

No. An identical twin just has identical DNA. But they have different experiences. Oh, what Mariah just said.

Rarebear's avatar

Okay, I’m off to see Deep Purple and Joe Satriani! I’ll pick this up tomorrow when I get back. I’ll let Mariah hold my torch for awhile.”

Mariah's avatar

@Rarebear Right, I think once the answer is found (if it ever is), the answer will be scientific and logical in nature. It just confounds me for the time being.

Eek, I’ll try!

Jeruba's avatar

I think of the brain as a physical organ and of mind as a process. There is a process because a living, thinking brain is not behaving the same as a dead, inert one. I think “mind” is the word we have given to our experience of that process. (The process itself is electrochemical, as I understand it.)

Our word in English, that is. And the fact that we have only one word for it suggests that we have allowed our experience to be very limited. Sanksrit, for instance, has several words for it and makes distinctions among those concepts that are difficult for Westerners to grasp.

The use of the word “mind” does not prove its existence as a separate thing; we talk about nonexistent things all the time and see no problem with that. But it does offer evidence of our experience of it as a separate thing.

InTheZone's avatar

@Mariah The OP states that the mind is a made-up construct. I think that the mind is a real thing that operates or is operated by the brain, which is a separate real thing. Software is a real thing, not something that is made up. It resides on a real medium, composed of real coding which is provided by the designer, and causes the computer to function in ways unique to it. Without the software, the hardware is without function. How can we think this isn’t real?

Jeruba's avatar

@InTheZone, I don’t quite buy your analogy. Here’s the computer. There’s the software. Ok, but until you run the program, nothing happens. Software is a set of instructions; it isn’t functionality, and it isn’t power. And there’s also the matter of data. Shut down your computer, and it still has its hardware and its software and its databases inside, but it isn’t doing anything.

I don’t much care for the brain-as-computer analogy because I think it leads us astray; but if we must use it, I would say that if the brain is hardware, the software is training, experience, learning, instinct, and all those other things that guide and inform thought and action (because of course the brain governs a lot more than just thought). Memory and sensory input supply the data. And mind is the process by which the brain operates on the data according to the instructions in its software.

RealEyesRealizeRealLies's avatar

Mind and Soul are identical phenomenon with different names from different disciplines. Same goes for Thought and Spirit.

Mind/Soul is manifest with the assemblage of numerous Thought/Spirit over the course of ones life, like bricks form a building.

Thought/Spirits of lust, greed, anger, compassion, empathy, kindness, charity… all assemble to create the Mind/Soul.

And yes, Mind/Soul is a separate agent from Brain, or any physical medium which represents them. Proof may be confirmed upon my leaving this thread. You all have access to the Thought/Spirit of my Mind/Soul simply by reading these words. But none of you will ever touch my brain. And after my brain is devoured by worms, you’ll still have access to the Thought/Spirit of my Mind/Soul just as if I were alive, yet silent.

Mariah's avatar

@InTheZone My view is, that which we perceive as our consciousness – our senses, thoughts, etc. arises from electrical and chemical reactions in a physical brain, and is not of a tangible enough nature to be considered separate from the brain. Just as software arises from electrical reactions on a circuit board and isn’t really a “thing.”

Perhaps the biggest difference between how you and I view this issue is semantics.

I have to go for now too, sorry @Rarebear!

RealEyesRealizeRealLies's avatar

For instance, we still have access to the Thought/Spirit of Jonathan Swift’s Mind/Soul after his brain is gone forever.

The republishing of Gulliver’s Travels allows more people to access his Thought/Spirit than ever before possible within his lifetime. But lets hypothesize that a few copies of his books are burned up in a fire. That doesn’t affect his Thought/Spirit in any way. Destroy 99% of his books, we can still access the Thought/Spirit of his Mind/Soul. We have no reason to assume that destroying all physical mediums would affect them any more than destroying one of them affects them. We’ve only lost access to them, and that’s all we can really claim.

ninjacolin's avatar

@InTheZone there are cases of identical twins who have an abnormal obsession without being around eachother and doing the same things.

As @Mariah explained, experience makes the difference. Just being on two sides of the same room gives two people separate memories of the event to work with. All the ittle differences add up to an entirely separate life.

Imagine if the two indviduals could somehow share the exact same perspective and memories with the exact same brain-hardware? Then they would be the exact same person.

RealEyesRealizeRealLies's avatar

Find an anonymous note in the sand. You are accessing a particular Thought/Spirit of the Mind/Soul of the original author. It doesn’t matter if that author is dead or alive. Access is absolutely equal.

It can’t represent something that doesn’t exist. And you can’t say they don’t exist because countless words from the past affect real change in the present. Change in the present could not be possible from something that didn’t exist.

RealEyesRealizeRealLies's avatar

Hey @Rarebear and @Mariah, I know you had to leave. But rest assured, in your physical absence, we have access to these limited Thought/Spirits of your Mind/Souls just as affectively as if you were here. Being here with us wouldn’t make any difference at all.

By the way, where is here?

@Rarebear wish I could be at that concert with you. Just thinking about it, well, let’s just say that in spirit, I am.

_zen_'s avatar

So wait, I’m a mindless, soul-less bitch?

I liked @Jeruba‘s thought, or is that not a thought?

Like many abstract things in life, starting with God and continuing through fear, love and hate – the mind, a terrible thing to waste, is just that – an abstract thing which we have given a semantic name for communicative purposes. No more, no less, than a soul.

Can I be convinced of what the OP has said in the details? Possibly? I think that is the main difference between our brain/mind and a computer’s software/hardware: for one, we made them and programmed them – not vice versa. And if you “ask” a computer what it thinks about its own software, it won’t answer “possibly” – it’ll answer whatever we’ve programmed it to.

To question is the mind.

The uniqueness of each individual, with no two thoughts at any given time, no two answers, no two questions, like snowflakes, ever alike. This uniqueness, is the mind.

Whether we give it one or five names, like in sanskrit, is less relevant. It’s like the Inuit have dozens of terms for snow – it just isn’t necessary for us – we all know what we mean when we say mind; whether it’s because you are out of your mind, mindless or mindful, or just in your right mind.

It’s semantics, communication, no more no less. But it isn’t just a big old organ with programming. It’s a fingerprint, of one’s ever-evolving thoughts, experiences and yes, soul.

kenmc's avatar

To me, it seems like an argument of semantics… which are usually wastes of time.

Dutchess_III's avatar

Well, bless my soul! How can you even ask such a thing?

You know…I guess we’ll find out, eventually. I know what I’d like to believe. I hang on to the idea that there is more than what we….can test. But….do dogs and cats have “souls”? If not, what would make us special that we do? Especially when so many humans are worse than animals.

I used to watch SOUL Train in the 70’s. Late night.

Enough with that.

The SOULS of my feet hurt, BTW. Not that the doctor would care, though. Humph

RealEyesRealizeRealLies's avatar

@Dutchess_III “do dogs and cats have “souls”? If not, what would make us special that we do?”

Humans can create new languages and words to describe new concepts at any time. Animals are stuck with the limited languages they are born with, never capable of expanding them.

I know that doesn’t seem connected to the discussion of Mind, but it’s actually what the whole thing is about. Principles of the word are what determines a beings level of consciousness, and therefor, mind.

jerv's avatar

@Rarebear Yes, I am, or at least close enough to make an analogy.

Notice how a hardware fault like a stroke or a head injury corrupts “data” just as a power spike/outage on a PC can corrupt it’s software? To continue the analogy, short-term memory is largely chemical and can be compared to RAM whereas long-term memory (more like a solid-state drive) is more structural.

Get drunk (alter your chemistry) and stuff may not stick around long enough to sink in so you won’t remember it more than a few minutes later; the cache never gets written to the drive. Get whacked in the head hard enough and you may forget stuff that you’ve known for years like your family, your childhood, or even your motor skills. Cut the power and, well, guess what. While it may not be an issue for short periods (it’s done intentionally in hospitals for some types of surgery), such a “power outage” will damage the brain structure as it starts to decompose. Too long and it won’t “reboot”; the person cannot be revived, or if they can be physically resuscitated, their mind is (either partially or completely) blank.

Coloma's avatar

I personally think animals are more aware, more in touch with their consciousness than humans. Humans have egos, animals do not. They react and act based on perfect, in the moment, and usually never wrong, action.

The bird does not sit in the tree obsessing about whether or not it will land on the next branch, it simply moves, and within it’s movement is perfection. The human would over think it’s actions til it finally fell out of the tree and broke it’s back. lol

Animals are brilliant forms of pure consciousness minus any grandiosity of self.

RealEyesRealizeRealLies's avatar

I am in the camp that believes human evolution ended between thirty and ten thousand years ago. That’s the very same moments when spoken language arose, to the point where written language was invented. At the point written language was invented, even as smoke signals and drum beats, human thought could transcend space and time. It couldn’t do that before written language.

At this point, human physical evolution is extinct. There are not enough environmental pressures for it to engage. I know we all think we’re gonna end up with big brains and little bodies in the future, but that’s just fantasy. No more human evolution. There’s no need for it.

Therefor our brains will not evolve either. But it cannot be denied that the human mind is evolving. Upon the invention of the internet, a rapid acceleration of the human mind is underway, right before our very eyes. This is taking place without the physical brain changing whatsoever.

RealEyesRealizeRealLies's avatar

We have different views about what consciousness is @Coloma.

Animals run their lives based upon cause/reaction.

Humans run their lives (or at least should) based upon thought/action.

Cognitive Studies uses no less than seventy different language tests to determine a patients conscious awareness.

A bee is only conscious to the degree his figure 8 waggle dance allows him to describe the location, quality, direction to to pollen. The bee is incapable of being consciously aware of a coffee cup sitting on the table. He has no language tools to describe such a thing with. The coffee cup is literally nothing to a bee. He is unconscious of it.

Qingu's avatar

To me the question is like asking “is there software, or is there just a CPU, hard drive, and memory?”

Which is to say, false dichotomy. Software runs on the CPU and memory and is stored on the hard drive. The mind runs on the brain and is stored in the brain.

RealEyesRealizeRealLies's avatar

This is another misnomer. Thoughts and Minds don’t spill onto the floor during brain surgery.

They are not “in” anything. Software is not “in” anything. They are immaterial agents. They don’t conform to notions of physicality whatsoever.

Memory, computer or human, is not like water in a bucket. It is not held. It is represented.

Qingu's avatar

Software and minds are patterns written on a substrate—a brain or a computer’s hardware.

But they’re not “immaterial.” They’re patterns. Similarly, the substrate is also a pattern. A brain is a pattern of interlocking neurons, which are in turn patterns of lipids, carbohydrates, proteins, and nucleic acids. A computer’s CPU is a pattern of silicon and copper atoms arranged just so to act like transistors.

RealEyesRealizeRealLies's avatar

Software is not a pattern. Software is a code. Codes and Patterns are polar opposites.

cockswain's avatar

TL, DR.

There’s this TED talk about the connectome that resonated with me.

RealEyesRealizeRealLies's avatar

Codes are language. They have a great deal to do with software and consciousness.

Patterns are determined by the fractal condition that chaos produces with cause/reaction.

Codes and language are determined by minds which engage thought/action.

There is a vast chasm between cause/reaction and thought/action.

Qingu's avatar

Semantics. A code is a pattern that can interact with one or more other patterns.

Your statement about chaos and fractals also appears to be nonsense. You’re not a mathematician, right?

ETpro's avatar

@Rarebear Great question. This is something I am fascinated by. I find no evidence that there is some supernatural presence running things within our head. I think your empiricist view is right. The mind, or “I“ness—the apparent homunculus that sits in our head and watches the movie our eyes project and pulls the levers to control what to do about what it sees, that’s an emergent phenomena that comes about when 150 trillion neural connections come alive and form self programming feedback loops designed for hundreds of different logical processing tasks.

These self-programming feedback loops are present in animals from relatively primitive ones up. The more sophisticated ones that are primarily specific to humans allow us to do things like make associations and form analogies. They facilitate learning. As social animals, we inherited from evolution an number of special self-programming processing units aimed at interpersonal skills and communication with group members. And we inherited a brain with a staggering number of axons and and even more humongous number of neural connections. From that came our emergent intelligence. And because we can form analogies so adeptly and can communicate about them, each generation can leave behind a record of ideas and conclusions that the next generation can build upon.

Pretty incredible stuff. One wonders how long till additional species on Earth reach full self awareness and begin down the same path. Some are close.

RealEyesRealizeRealLies's avatar

No I’m not a mathematician @Qingu.

Codes may be written as patterns. Any song chorus will confirm this. But patterns in and of themselves, are the polar opposite of codes. And though the “statement about chaos and fractals appears to be nonsense”, that doesn’t mean it’s nonsense. It only means you don’t understand my statement.

Do you deny that fractals are patterns produced by the cause/reaction of chaos?

Do you deny that codes and language are produced by the thought/action of mind?

RealEyesRealizeRealLies's avatar

Do you deny that code is the representation of a thought from its author?

When pushing pixels in photoshop, the only physical tools that exist are the hardware of the computer. But the actual software, is nothing more than the combined thoughts of all those guys you see listed when clicking “about photoshop”. We use their thoughts as tools.

RealEyesRealizeRealLies's avatar

@ETpro “I find no evidence that there is some supernatural presence running things within our head.”

No one is suggesting supernatural. What I propose is perfectly natural.

@ETpro “that’s an emergent phenomena that comes about when 150 trillion neural connections come alive and form self programming feedback loops”

That would be supernatural. For if we are nothing more than a current assemblage of stardust, it suggests that ultimately the dumb mute cosmos can somehow enjoy a movie.

Qingu's avatar

Oi. I’m not sure I’m willing to go down another one of your rabbitholes, @RealEyesRealizeRealLies.

But software is not “thoughts.” That is nonsense. It is a pattern of electronic signals. The pattern is designed to have certain effects and work in a certain way with other hardware, and other software.

The words in a book are also not “thoughts.” They are written symbols. They would be meaningless without a mind to interpret them, but that doesn’t mean they are a mind.

Similarly, DNA is a code; it is not made of thoughts; it does not even require a mind to understand. It simply works like the pattern of a key. It’s a pattern (of nucleotides) that works in a specific way with another pattern (proteins).

RealEyesRealizeRealLies's avatar

The electronic signals are a representation of thought. You did use the word “signal”. Electricity doesn’t signal anything unless someone uses it as a medium to transmit a signal upon. Can’t do that without thought.

Every code is a representation of, or manifestation of thought. No exceptions.

RealEyesRealizeRealLies's avatar

If you believe that electricity can signal anything without a thought behind it, then you believe that the dumb cosmos can signal. What is it saying to you?

RealEyesRealizeRealLies's avatar

@Qingu “The words in a book are also not “thoughts.” They are written symbols. They would be meaningless without a mind to interpret them, but that doesn’t mean they are a mind.”

The symbols represent the thoughts of the author. The medium is not the message. It only represents the message.

Qingu's avatar

Thought is not required for signals. You can make a genetic algorithm that “dumbly” evolves signal processing.

RealEyesRealizeRealLies's avatar

As I said, “or manifestation of”... A.I. and computer science are rife with examples of code which can be authored to re-author itself. But it always begins with the thought of an original programmer. It would never happen without it.

RealEyesRealizeRealLies's avatar

Books don’t evolve unless a mindful editor puts an additional thought to it.

RealEyesRealizeRealLies's avatar

@Qingu “DNA is a code… It simply works like the pattern of a key. It’s a pattern (of nucleotides) that works in a specific way with another pattern (proteins).”

The genome is sometimes called a “blueprint” by people who have never seen a blueprint. Blueprints, no longer used, were two-dimensional, a poor metaphor indeed, for the linear and digital sequence of nucleotides in the genome. The linear structure of DNA and mRNA is often referred to as a template. A template is two-dimensional, it is not subject to mutations, nor can it reproduce itself. This is a poor metaphor as anyone who has used a jigsaw will be aware. One must be careful not to make a play on words.”—
Hubert Yockey, Information Theory, Evolution, and the Origin of Life

RealEyesRealizeRealLies's avatar

“Information, transcription, translation, code, redundancy, synonymous, messenger, editing, and proofreading are all appropriate terms in biology. They take their meaning from information theory (Shannon, 1948) and are not synonyms, metaphors, or analogies.”
Hubert Yockey, Information Theory, Evolution, and the Origin of Life

There is no mention of patterns or keys in Yockey’s description of DNA.

RealEyesRealizeRealLies's avatar

Agreed, let’s not go down the rabbit hole of G authoring DNA. I’m trying very hard not to go there. But this discussion does have a great deal to do with linguistics. Talk of code and language are very appropriate. I’m sorry, but I cannot let them be conflated as patterns and keys. That I will defend against.

Uberwench's avatar

The mind is something the brain does, just like running is something your feet do. Both exist, one as an object and the other as a process.

RealEyesRealizeRealLies's avatar

This discussion points to the noun of mind. Not the verb.

ETpro's avatar

@RealEyesRealizeRealLies There is most certainly no evidence of a natural spirit. Nothing can be detected leaving the body at brain death. It makes you wonder how come the guys from Roto-Rooter™ in the TAPS guys on the Ghost Hunters show are able to detect Ghosts with EM meters and IR cameras and all sorts of other ordinary instruments long after someone has died.

No, the brain starts working in the womb in rudimentary fashion and becomes more powerful as it grows and learns. There is no point where some detectable, natural force from outside inhabits it to kick start self awareness. There is also no point where such a natural force can be detected slipping away from it upon brain death.

If I were to assert that the human brain suddenly sprung into existence fully formed with all 150 trillion neural connections arranged exactly as needed to give it all its amazing processing powers, that would certainly be an appeal to the supernatural. But please understand I am asserting no such thing. Instead, the processing capacities have been evolving in ever more complex life forms over a period of about 3.5 billion years according to the available fossil record.

That is such a staggeringly long time for neural processing ability to evolve that it’s incomprehensible to our brains without calling on analogy. One of the best I have heard turns to a length measurement in the English system, the yard (3 feet, or 36 inches). It is said that the yard was standardized as the distance from King Henry I’s nose to the tip of his fingers on his outstretched arm. If we accept that, and imagine that one yard represents all of evolution, then a single light brush of a nail file on Henry’s index fingernail would erase all of human history. What neural connections on nerves and axons do has been in development for a VERY long time. We’re just the culmination (at this time) of that development. There are numerous other higher mammals able to form rudimentary analogies, communicate and exercise social skills, and even pass on communal knowledge. Elephants, orcas, dolphins, chimpanzees and bonobos are a few that come to mind. It isn’t as if our capabilities suddenly poofed into existence fully formed a few million years ago.

Coloma's avatar

@RealEyesRealizeRealLies

Yes, I guess we do.
Brain ‘consciousness’ aside, I go with Eckhart Tolles definition that everything, human and animal is pure consciousness, and acting from pure awareness minus thought, is pure consciousness at work.
No thought = right action, born of pure consciousness.
Sort of like saying getting out of our own way.
But that’s another question.. ;-)

Uberwench's avatar

@RealEyesRealizeRealLies I don’t think the noun of mind makes sense except through the verb, and I’m free to answer as I wish.

RealEyesRealizeRealLies's avatar

@cockswain “this TED talk”

His analogy of water as thought which forms the shape of the bed as connectome is exactly the same as mine of many thought/spirits building a mind/soul like bricks in a building.

@ETpro “the brain starts working in the womb in rudimentary fashion and becomes more powerful as it grows and learns.”

Fits perfectly with the analogy above. This is actually beginning to change my views on abortion. Once I believed, and still tend to, that a human exists at the point of a full genetic code, within hours of conception. Now I am forced to consider that a human being may not be fully human until some degree of Mind/Soul has arisen through and assemblage of Thought/Spirit. That may take months, as baby becomes capable of recognizing mothers singing within the womb. I don’t know where I stand on this currently, but I do agree that humans don’t arise with a fully developed brain capable of expressing thought.

@Coloma “Eckhart Tolles”

I have many problems with Tolle. I firmly believe he has bastardized eastern teaching, homogenizing them for western consumption. His concepts of the “Now” are completely wrong as far as I’m concerned.

@Uberwench “I’m free to answer as I wish.”

Was that freedom discovered with the noun or the verb of mind?

Coloma's avatar

@RealEyesRealizeRealLies

Well….that is your opinion and you’re entitled to it, of course.
There are many good works, and he has simply offered up a sampling of simplicity for many that might never read the Tao, or study non-duality philosophies like Adviata Vedanta or any of the other eastern philosophies.
His works have value for many, don’t knock him, unless you have written something better to assist the masses. lol

wundayatta's avatar

You’re talking semantics here. How do we define a word? I think the term “brain” is pretty clearly defined. “Mind,” not so much. The brain is a thing. The mind is a metaphor.

I think “mind” is a helpful metaphor even if it can’t be defined very clearly. It’s kind of a gestalty thing. It’s another way of talking about what you are thinking about (what’s on your mind). There should be no feeding frenzy about the term. It’s just a metaphor.

Qingu's avatar

The fact that people call DNA a “blueprint” does not mean it is actually a blueprint drawn up by a conscious person.

This is what I meant when I said I have on interest following you down your rabbit holes.

MRSHINYSHOES's avatar

@Rarebear Of course it’s a circular statement! Just like 1 and 1 are 2 and 2 is 1 and 1. Doesn’t mean it’s wrong. On the contrary, it rings true.

RealEyesRealizeRealLies's avatar

@Qingu You’re creating the rabbit hole by refusing to acknowledge what DNA actually is. Call it what you want… But it is NOT a blueprint, not a template, not a pattern, and not a key.

It is a four letter alphabet which encodes information into a three letter alphabet to create you. It’s called a code for a very specific reason. It is not called any of your alternatives for very specific reasons. It’s a full blown communication system with syntax, semantics, redundancy, error correction, and noise reduction. Please stop creating confusion by claiming it is something other than what it actually is.

Again, out of respect for @Rarebear, as we both suggested earlier, could we please avoid getting into the debate of codes requiring authors? The subject of language was brought up only because it is crucial to any discussions of mind and consciousness, not so we could derail this thread with a debate about origins.

Rarebear's avatar

Back. Frakking awesome concert. Since it’s after midnight I’ll read responses tomorrow.

Ghajini's avatar

Interesting question-
There are so many things about brain that we don’t know. Consciousness, conscience, altruistic behavior…
Anyways, you have a body that works when you are alive- but doesn’t do much after you are dead- so similarly, a brain that is dead can’t do anything.

Life is the distinguishing factor according to me- A brain that is OFF is just a brain- a brain that is ON is brain and mind.

Dutchess_III's avatar

@RealEyesRealizeRealLies There are some animals who can communicate with an amazing ability. Dolphins, gorillas. There are animals who are capable of “thinking outside of the box,” figuring out solutions to problems in their minds. Do they have souls?

flutherother's avatar

There is an interesting lecture here that touches on this question.

dannyc's avatar

The brain is the organ, the mind is the information. Electric, metaphysical, spiritual or however you wish to describe said data. And Satriani and Deep Purple will blow your mind..

flutherother's avatar

Self edited

RealEyesRealizeRealLies's avatar

@Dutchess_III To the degree that I associate Minds and Souls as Mind/Soul, and suggest that agent as something which is built over time through experience, and the descriptive authoring of those experiences, then yes, animals can achieve a certain degree of Mind/Soul.

I understand my philosophies are distasteful to most. But from my research, I cannot claim that we are born with souls any more than we are born with minds. Mind/Soul is created throughout our lifetimes, brick by brick. Thought/Spirits are the bricks which build a Mind/Soul.

Most people discuss the subject of consciousness as something that is either on or off. I can’t accept that. I believe there are varying degrees of consciousness, and the level of consciousness is directly relevant to the level that a life form can use language tools to describe physical and theoretical phenomenon. I firmly believe that a person who speaks multiple languages is more consciously aware of his environment (and his relationship to it) than a person who speaks only one language. They have more descriptive tools at their disposal.

A well read person is more consciously aware than an illiterate person. An adult is more consciously aware than an infant, simply because of the greater selection of language tools available to describe the phenomenon they experience.

Thus, when an animal demonstrates a capacity to solve an abstract problem, we should consider that his ability to achieve such a feet is directly related to how efficiently he can envision the problem, and plan a specific procedure beforehand which satisfies the original vision. All plans are made with code (language). The animal is literally authoring an instruction manual which details the events necessary to accomplish the task. He may author one simple instruction at a time, but he does it by encoding the desire of a Thought/Spirit (hunger, greed, fear), and thereby increases the potentiality of his Mind/Soul. Additional Thought/Spirits expand the Mind/Soul potentiality much in the same manner that additional bricks expand the potentiality of a new West Wing at County Hospital.

Dutchess_III's avatar

I’m trying to understand @RealEyesRealizeRealLies…so…a one day old baby who hasn’t learned a language and has had no time to build through experience wouldn’t have a soul? I’m not being sarcastic. I like your philosophy. Just trying to understand.

RealEyesRealizeRealLies's avatar

Language manifests in subtle ways. Any time an image/object association is connected, there be the fundamentals of a language at work.

At the point when baby in the womb associates mothers singing with comfort, then an image/object relationship has been established. The baby has become more consciously aware than she was before the association was made.

Dutchess_III's avatar

And…animals don’t do the same thing? I mean, I don’t know.

RealEyesRealizeRealLies's avatar

I don’t know of any research which suggests that animals sing to the babies in their wombs. But it very well could be that the babies begin to hear sounds and feel consistent sensations which they associate with certain auto responses. I have no idea.

GracieT's avatar

When I was an infant, after my parents adopted me, my mother read to me. No matter what she was reading. Advertisements, children’s books, road signs, even the book she was currently reading herself. She did it so that I would start to recognize the language and how it was used. It was largely because of that that I began to read voraciously, and am so in love with language.

Coloma's avatar

Well…it has been proven that stimulating environments increase brain size and, I have long believed, and seen this to be true in my animal experiences.
My 13 yr. old Chinese goose knows about 20 words, 6–7 commands, hand signals and, he learned to turn the hose nozzle on with his beak and put it in his pool. lol
He flings it out when it is overflowing, but, he doesn’t turn it off. haha

I am sure domestic animals in utero pick up on their mothers emotional state of mind as well as sounds in the environment they will be born into.

As far as ‘souls’...well, I believe all energy, consciousness, is part of the integral whole, so, an animals essence would certainly be included IMO. Consciousness is consciousness and who are we to say that an animals consciousness is somehow inferior to ours?

Dutchess_III's avatar

I never sang to my kids when I was pregnant….I’m not a singer.

@Coloma If a stimulating imprisonment increases brain size, your brains should be oozing out ya ears!

@GracieT We had the same kind of mom!

Coloma's avatar

@Dutchess_III

LOL, well..not imprisonment, stimulation. Like lots of things to do and explore.
Studies have shown that baby rats kept in empty tanks with no mental or physical stimulation have smaller brains than their jungle gym, pellet pushing, maze running peers.

Rarebear's avatar

Wow, thanks for all the great responses.

In reading through I’m seeing two points of view. One point of view is shared by the empircicists, people like Mariah, ETPro and myself. ETPro put it the best:

“that’s an emergent phenomena that comes about when 150 trillion neural connections come alive and form self programming feedback loops designed for hundreds of different logical processing tasks.” Exactly. It’s what happens when you put that many neural connections together, that conscious emerges and a sense of self also emerges. But it’s not separate from this.

The other camp, which seems to be the majority, is personified by people like Realeyes. This camp believes that the “mind” (and by extension the “soul”) is a separate phenomena than the “brain”. It’s something extra and unique.

ETpro's avatar

@Rarebear The dualist concpet is the intuitive one. If you come at what is “I“ness in the lack of any evidence—which is what mankind did throughout most of its history and what most of mankind still does when it comes to advanced research on the human mind and on neural networks for Artificial Intelligence— then the dualist answer just inherently “feels” right. It is also invoked by most world religions. Thus it is incredibly hard to shake. While I doubt dualism’s truth strictly through an intellectual exercise, it still “feels” like there’s a Mind/Soul in there turning the cranks of the nervous system, making the decisions, being “me”.,

MRSHINYSHOES's avatar

I think when people overanalyze things and start splitting hairs, then they begin to LOSE their minds! Hehe. ;)

RealEyesRealizeRealLies's avatar

For another perspective, completely non religious, I invite you to consider the mind of Terrence McKenna. Tryptamine Consciousness is one of numerous dissertations he discusses language based consciousness. It’s deeper than my description suggests. He believes there are realms of reality, “one quanta away” where language is not a tool, but a method of existence. In these realms, language is not heard, but it is beheld visually with the minds eye. I don’t expect this one podcast to make much sense to anyone who hasn’t studied McKenna and his philosophies. But careful with McKenna, he’s quite addictive. The Psychedelic Salon has countless podcasts on his theories and research, which are vast and varied.

I’ll leave it to you to decide if this is science or science fiction. But the alien life form which McKenna was in contact with told him the following:

“There is nothing unusual about this. Man’s conceptions of organized intelligence and the dispersion of life in the galaxy are hopelessly culture bound. The galaxy has been an organized system for billions of years. Life evolves under so many different regiments of temperature and pressure that the trick is to know that contact is being made at all.”

We couldn’t recognize an alien life form even if it sprouted up in our own backyard. We shouldn’t suppose that they are all humanoid, or that they are even physical entities.

Rarebear's avatar

@RealEyesRealizeRealLies When non-physicists start throwing the words “quantum” and “quanta” around, I get very skeptical. Quantum mechanics is hard enough for physicists to understand, let alone philosophers.

RealEyesRealizeRealLies's avatar

He used the term metaphorically to designate a reality that was very close, right under our noses. It was not used in any specific scientific manner. I should have clarified that, sorry.

Coloma's avatar

My latest discovery is Nassim Harameins ” The Black Whole”.
He is a Swiss geometricist that studies hyperspace, theoretical physics and quantum mechanics, along with ancient civilizations.
His highly acclaimed, award winning paper “The Schwarschild proton” is “mind” boggling to say the least.

A good case for the consciousness/spiritual/physics connection.

We ARE Black “Wholes.” ;-)

Qingu's avatar

@Coloma, there is no such thing as a geometricist. And I am having trouble finding any awards for this Schwarschild proton that come from people who actually know anything about quantum physics.

Sounds like new-age bullshit. No offense. If you want to learn about consciousness and mathematics from, you know, people who actually know what they are talking about, I’d suggest reading Douglas Hoffstader’s “I Am A Strange Loop.”

Qingu's avatar

And regarding the Schwarschild proton, one of the first things I found on my Google search was this website.

Choice quote:

So Haramein introduces us to the Schwarzschild proton. This is a black hole with a mass of 8.85×10^14 gm. In plain English, this is 885 million metric tonnes.
...
The paper begins with the suggestion that a real proton may be considered to be one of these. To see if this is workable, let’s compare his model with with what we already know about protons.

Mass of an actual proton: 1.67 trillionths of a trillionth of a gram
Mass of Schwarzschild proton: 885 million metric tonnes

These aren’t particularly close.

How does Haramein deal with this discrepancy from reality?
He doesn’t.

Simone_De_Beauvoir's avatar

Yes, there is just brain but as others have said, we don’t know much about the brain and how many of the multiple parts actuall work together to produce much of what amazes human beings about human beings. To me, brain and mind are the same thing and we know little about either. Saying it’s all about biology doesn’t mean we know what it’s about.

RealEyesRealizeRealLies's avatar

@Simone_De_Beauvoir

Is there any fallacy in claiming they are the same, yet speaking of them as an either?

Is there any fallacy in claiming what it is (to you), yet saying “we know little”, “we don’t know much”, and “doesn’t mean we know what it’s about”?

Simone_De_Beauvoir's avatar

@RealEyesRealizeRealLies I don’t see it as such a big fallacy, if any. After all, I am one person but I am a mother and a lover, let’s say. And certainly no fallacy when it comes to your second proposition. You know little of me but I am.

Response moderated (Personal Attack)
Rarebear's avatar

@Qingu How could she possibly NOT take offense when you say it’s bullshit and then say “no offense”? I’m skeptical of the claims of the Schwarzchild Proton, but I reserve judgment until people smarter than I can work through the math. I checked all the usual skeptic blogs and it hasn’t hit them yet unless I have missed it. And I’m certainly skeptical of the spiritual claims made by it. But that said, I still try to be respectful.

Coloma's avatar

@Rarebear

I respect and appreciate YOUR respect ;-)

Qingu's avatar

@Coloma, no, it’s bullshit.

If you’re interested in why, click the link I posted. I would also be happy to explain why it’s bullshit based on my own knowledge of quantum mechanics, though I’ll note you haven’t really explained what you think he’s correct about.

And I’m sorry but you don’t get to play the “you don’t know anything about science” card with me.

And I do mean this with respect. Everyone gets taken in by bullshit and snake oil every now and then. But don’t mistake my respect for a willingness to tiptoe around pointing out the fact that bullshit is bullshit.

Qingu's avatar

@Rarebear, the claim he makes—that the proton is actually a black hole—is so nonsensical that I don’t even know where to start (neither did the blog I linked to).

I mean, protons are baryonic structures made of 3 quarks. This has been known for years. It’s the exact same model that explains what a neutron is. It’s the exact same model we use to predict how nuclear reactions work, in technology that we have built and used.

A black hole, by definition, is basically like a particle. It only has particle-like properties: spin, mass, and charge. It has no internal structure. This is the whole deal about black holes “swallowing information.” If a black hole contained 3 quarks, we would never be able to know this fact, because those three quarks cannot interact with the universe outside the black hole.

So it is literally nonsense to say that a proton—which we know for a fact has an internal structure of quarks and chromodynamics—is a black hole. It’s gobbledegook. It’s worse than the filler lines on Star Trek, because at least Star Trek writers know something about quantum mechanics. It is scientific-sounding nonsense designed to hoodwink people who are ignorant of science.

Or, to put it another way, it’s bullshit.

And I haven’t even gotten into the claims that this guy has apparently made about magic pyramids and space aliens. To put a finer point on it, that’s new age bullshit.

Or who knows? Maybe everything every physicist has known for 80 years, and all of the ideas behind our working technology and nuclear weapons, is completely wrong, and this one guy on the Internet with a cult-like following making spiritual claims, is right.

And maybe I’m a Chinese jet pilot.

WasCy's avatar

@Rarebear

I have to go with @Qingu on this. Disabusing someone of nonsensical ideas is a hard thing to do. There’s no easy way about it, especially when the nonsense of the idea has been pointed out, and yet the idea is clung to. Part of the problem is the somewhat natural tendency that people have to self-identify with the nonsense. I know, because I’ve had my own ‘conversions’, for lack of a better term.

@Qingu was not dismissive of @Coloma that I could see, but certainly of the silly idea that she holds onto. Maybe several silly ideas. And that probably feels to @Coloma like he’s dissing her, but I don’t think he is. I expect that if he thought that she was the originator of the nonsense or was attempting to profit from promoting it or controlling others, then he would have been considerably harsher – and more personal.

There’s a lot of New-Age (and age-old) bullshit around. There’s no simple, gentle, tactful – and effective way – to dismiss it all, but it has to be dismissed if anyone is ever going to learn anything more fact-based.

Coloma's avatar

@Qingu
And..” your own knowledge” is what…?
Fine, you are the self appointed guru of all, and therefore no new materiasl can enter your cranium?
No room for any expansion or new data to enter, because @Qingu SAYS it’s bullshit!

Well…I don’t see YOUR name behind any published theories!

Qingu's avatar

I’d rather not divulge my actual identity on these intarnets, but I do edit science articles for a living.

But I’m not appealing to my own authority to say that this guy is full of shit (that would be, more accurately, an appeal to the _entire scientific community of physicists). I’m saying that I know more about quantum mechanics than you do. Which is why the “you don’t know science so you can’t prove me wrong” trick isn’t going to work on me.

Maybe I’m wrong. @Coloma, Why don’t you explain what you think a proton is, and why.

Coloma's avatar

@Qingu

You are answering a question with a question.

Editing science articles does not make you a scientist. That’s like saying wine tasting makes you a vintner. Heh!

My input is dust and yours is dustier. lol

Watch the film. You can do the math with Haramien, step by step.

Qingu's avatar

Are you a scientist?

And if you’ve seen me interact with people on here before, you’ll know I have a policy of not “just click on the link” with people who outsource their arguments instead of making them in their own words.

Why can’t you tell me what you think a proton is? Isn’t that, you know, what this guy’s amazing theory is supposed to be about? Are you really so utterly convinced that this guy is right but completely unable to give a rudimentary explanation of what he’s right about?

Rarebear's avatar

You don’t understand what I’m saying. I’m not disputing that it’s most likely bogus. I’m talking about the tone of his comment to Coloma. That’s all. When you say, “What you just wrote is bullshit, no offense.” The “no offense” is supposed to take the sting out of an ad hominem attack. I happen to agree with you guys, but there are much nicer ways to debate.

Coloma's avatar

@Qingu

www.theresonanceproject.org

Read, Make up your own mind. I am not a scientist, I am simply a seeker of knowledge, and, an open minded individual.

I don’t have the time to blather on in egoic debate..read and come to your own conclusions.

It still stands that he has been recognized for outstanding achievments in Physics,and, you have not.

Qingu's avatar

Why do you want me to read this guy when you can’t even tell me the first thing about what this guy has said?

If you can’t tell me what a proton is, how on earth are you even in a position to judge whether or not he’s bullshit?

And no, he has not been recognized by the scientific community. Real scientists don’t run websites hawking for-profit DVDs about their research. He’s a fraud.

Qingu's avatar

And that website is promoting a cult along the lines of Scientology. How much money have you given them, @Coloma?

Response moderated (Personal Attack)
Response moderated (Off-Topic)
Qingu's avatar

@Coloma, this really isn’t about me. I’ve also been duped by pseudoscience.

The guy you are referring to is clearly a fraudster who, like L Ron Hubbard, has created an elaborate scientific-sounding edifice upon which to build a cult. He sells membership to this cult for $10 a year, and I see a number of other products on sale on that website. You owe it to yourself to not be duped.

WasCy's avatar

@Coloma how do you know that he @Qingu hasn’t got a Nobel Prize for Physics? You’re assuming that he doesn’t simply because he hasn’t crowed about it “in egoic words”. Based on the body of his science-based responses in Fluther, his word on the topic carries more weight than most, I think.

Qingu's avatar

Scientific credentials are not necessary to understand that the Resonance Project is a fraudulent cult.

That said, there really is no reason to assume that I have a Nobel prize… I’m not even a scientist. It’s my job to stay informed about scientific matters and to at least know what I’m talking about in a very basic sense, but I would definitely defer to some of the Flutherites who I suspect are actual scientists.

Rarebear's avatar

@Qingu That’s why I was looking through the physics and astronomy blogs to see if it’s been mentioned.

Qingu's avatar

Well, here’s a good counterexample.

A while ago I was quite taken by an article about the nature of gravity, claiming that gravity does not exist at a fundamental level but rather emerges from the macro-scale properties of entropy. I didn’t understand most of the math in this article but the idea seemed (still seems) elegant and reasonable to me.

Verlinde’s idea, like Haramein’s ideas, is contrary to “mainstream” physics and would, if true, overturn a lot of what we think is true.

Unlike Haramein, however, Verlinde:
• Isn’t hawking DVD’s or group memberships
• Hasn’t set up a cultlike website
• Hasn’t extended his idea about the physical properties of gravity (or protons) with broad-reaching implications dealing with ancient civilizations and human society
• Has actually been buzzed about in the scientific community
• Has written his paper in a way that, at the very least, interacts with physics as we know it, as opposed to being complete gobbledygook

There are a lot of out-there claims in the scientific community, and there’s a good chance that at least some of them have merit. But many claims are bullshit. And, I mean, if someone is aggressively promoting a scientific claim that you don’t understand, the first thing you should ask yourself is “is this claim being used to support what sounds like a new age religion?”

ETpro's avatar

@RealEyesRealizeRealLies Thanks for posting that link. I am all the more detemined to meet the Machine Elves before I die.

RealEyesRealizeRealLies's avatar

Glad you enjoyed it @ETpro. Perhaps we’ll visit the Elf Machines together one day. Cool if we could convince @Coloma, @Rarebear, and @Qingu to join us.

I really appreciate McKenna’s perspective on this. Rather than approach the Logos on bent knee as “Dorothy before OZ”, we should meet it with dignity, standing on our feet.

ETpro's avatar

@RealEyesRealizeRealLies I would truly love to be among intelligent and exploration friends. Have you looked into the least costly venue where one can legally do it? Perhaps it’s time to charter a new religion.

RealEyesRealizeRealLies's avatar

Yes I’ve researched it. $1500 gets it done with a tourist Shaman in the Amazon basin. I’m looking into it deeper. I won’t settle for anything less than a Shaman with a family heritage, willing to demonstrate the process, rather than simply pour the kool-aide. I’ll keep you posted.

RealEyesRealizeRealLies's avatar

Let me rephrase. A tourist Shaman is not good enough. They are dangerous and ill equipped to handle the crowds they draw. That scenario is a facade. I will not settle for it.

There are other ways, much cheaper, genuine, one to one. A Shaman with a family heritage of mixing the brew is the only thing I’ll accept. It’s an entirely different experience. More info at Deoxy.

ETpro's avatar

@RealEyesRealizeRealLies Warning understood. A tourist shaman wasn’t what I had in mind either. And yes, please do keep me posted.

Response moderated (Personal Attack)
Qingu's avatar

@Coloma, I did look at the evidence presented for his work. It’s gobbledygook.

And you can’t even explain what his work is about. I’m still waiting to hear what you think a proton is.

RealEyesRealizeRealLies's avatar

I hope she PM’s you on that @Qingu. @Coloma‘s thoughts on what a proton is has nothing to do with…

“Do you think there is a “mind” or is there just “brain”?”

Rarebear's avatar

I had a discussion with Auggie. We’re moving this to social—otherwise ⅔ of the thread will be modded as off topic.

augustlan's avatar

[mod says] Moved to Social. Most answers have been restored. Flame off, folks.

ETpro's avatar

@augustlan Mods rock… sometimes :-)

RealEyesRealizeRealLies's avatar

I recently posted a question about group hypnosis describing a new personal experience with the feeling of “the world didn’t exist any longer, and I was floating through space and time.”. @PluckyDog suggested Photosensitive epilepsy of which I’m mostly in agreement with. However it came about, and whatever description I provided about myself and others, an equal description would be to describe a “mindless” state. I would also describe the others as mindless too, or at least they looked that way, thus I described them as zombies. More details available in my other thread.

How I could “feel” something without a mind to feel it with is beyond me. And if someone else described the same for themselves, I’d probably debate against the assessment, using my code rap as support against it. Perhaps not. I just haven’t considered the episode enough to really comment intelligently. The event affected me. I’m just not sure how… yet.

If I were debating against it, I’d ask: Could you have formed a sentence during the event?

Answer: I have no idea. It never crossed the mind I felt was missing.

See the paradox?

To even get there from here… A mind would be necessary to desire the action of crafting a sentence. In doing so, a thought must form to embody the symbols with meaning. Not to mention a mind would be necessary to understand the suggestion from someone to write the sentence… in that a mind would need to translate the sound symbols from the person who asked.

Turning the focus on the other 200 Zombies in the room, lets run a hypothetical experiment. I’m going to get really deep with this, so tune out now if you’re not going to put some equally deep thought into it. I’m requesting what your prediction of outcome would be for this hypothetical experiment. I’ve never heard of this being done, intentionally.

CONDITIONS: Highly visual movie such as Transformers III.
The mind is engaged on many levels, with the obvious being story line, sound track, and highly potent visuals which require a tool (3D glasses). But the mind is actually engaged on other more subconscious levels. In a sense, a doorway of desired potentiality is opened up the very moment one decides to see the movie, buy the tickets, smell the popcorn… and it is engaged even deeper when putting on the 3D glasses. That’s like strapping into the cockpit, fully knowing that a ride is about to commence, as the bar comes down over the waist in a roller coaster. We get ready for it. The second definition of Apprehension is added to the equation.

TEST: At a predetermined time, let’s say 1 hour into the movie, as everyone is fully immersed within the most visually stimulating scene supported with the peak of dynamic sound track at a critical moment in the story line… BAM! Shut the movie off dead stop and project this sentence on the screen black on white:

YOU ARE THE SUBJECT OF A MIND EXPERIMENT
Please reach under your seat to find a notepad and pen. Describe the events that you were just watching at the point where the movie shut off. Upon finishing, press the red light button in front of you.

PREDICTION: My question to you… How long do you think it will take from the moment the movie shuts off until the red light button is pressed?

Basically what I’m asking is how long do you think it would take for people to come out of the immersion they were fully engaged in, understand the request (be able to language again), and complete the task?

Let’s please assume that people would cooperate and not throw their popcorn at the screen in anger. Please assume this, for the sake of the hypothetical.

Would it make a difference if the screen didn’t make a request with instructions, but instead just flashed !!!FIRE!!!? How long would it take for people to understand and choose a course of action? Would it remain civil?

And one more question… If the state is truly mindless, as it feels to me, what is the condition of the Mind/Brain relationship during this event? You see what I’m getting at? If the two are actually capable of being separated, how long does it take to re-engage the connection between them? That’s slanted, I know. Perhaps you don’t accept they were inseparable to begin with. In that case, please describe how you think this works.

Just moving the discussion along for those as interested in the subject of mind and brain as much as I am.

Rarebear's avatar

@RealEyesRealizeRealLies Great answer.

Your answer seems to presuppose that there is a difference between the brain, which is a made of meat and the mind, which is the thinking and reasoning portion. I’m saying that it’s just the hunk of meat that is doing that work. Everything else you are saying is true, but I don’t see any duality between the mind and the brain. Mind is brain. The abstract thinking that you speak of is a function of the very complex neurochemical and electrical mechanisms.

GracieT's avatar

Like @RealEyesRecognizeRealLies mentioned flashes of light can trigger seziures, but my Recreation Therapists in the hospital said that for any kind of seziures flashing lights and chocolate can be triggers. The type of seziures I have are Simple Partial, so I retain memory of them. Unfortunately, though, all epileptic medicines are drugs that “relax” the CNS. A side effect of all drugs of that type is that they have the potential of “relaxing” the memory. Period. One of them, my Neurologist told me, Topamax, is called “stupamax” by many Neurologists for just that reason.

RealEyesRealizeRealLies's avatar

What is the state that the mind of Abraham Lincoln is in right now?

The Gettysburg Address represents one of his Thought/Spirit now just as well as it did back then. All thirteen versions of it, each slightly different from one another. The potency of thought, which is the Gettysburg Address has actually grown in potency over the years.

If Lincolns brain is gone for good, then how is it that we can access a Thought/Spirit of his currently? And how could that Thought/Spirit still exist without a Mind/Soul for it to expand? It is expanding… How could something that doesn’t exist, expand?

Rarebear's avatar

Wait what? We can access the Gettysburg Address because it was written down. Nothing spiritual about that.

RealEyesRealizeRealLies's avatar

Just like our comments here are “written down”. But these are our comments, not our thoughts. These comments represent our thoughts, just like the “writing down” of the Gettysburg Address is making a comment about a thought.

These comments are not equal to thought. Where is the Thought/Spirit that the Gettysburg Address represents?

Thought is not reducible to electricity and photons any more than it is reducible to electrochemical neural connections.

RealEyesRealizeRealLies's avatar

I noticed that under the Generalized category of Epileptic Seizures, the first label given to a double hemisphere attack is called Absence, which ”triggers a short lapse in consciousness.

What is Absent? The brain is still there, but Elvis has left the building.

Ink + Paper + Glue do not make a book. Thought is a separate and necessary ingredient. Without it, a book is just a rock.

Uberwench's avatar

@RealEyesRealizeRealLies It was discovered by the verb. When “mind” is a noun, it stands for the process. The only physical object involved is the brain (and, by extension, whatever it needs to function).

RealEyesRealizeRealLies's avatar

A noun that stands for a process has “ation” attached to denote it as a process. Like inform-ation.

I’ve never heard of the word mind-ation before.

If the noun stood for the process, then the verb wouldn’t be necessary. That’s what the verb is for.

Uberwench's avatar

@RealEyesRealizeRealLies The problem is that the language around this phenomenon evolved when people had mistaken ideas and thought that the mind was an object. So unfortunately, you can’t expect the linguistic situation to conform to reality.

RealEyesRealizeRealLies's avatar

The debate of mind/brain duality is as old as history. To claim “mistaken ideas” shows little regard for the depth of the subject. The philosophers of earliest language were very precise in their use of words. So much so that modern linguistics still follows Greek and Latin form for crafting new words and many science terminologies.

You can disagree with me, no problem. But please contribute something more than just “mistaken ideas”. Refute something I’ve said or point out a flaw in my argument in favor of dualism.

Uberwench's avatar

@RealEyesRealizeRealLies I haven’t read the entire thread. I’m just continuing the conversation you started with me earlier. The question asked for my views, and I have given them.

InTheZone's avatar

@Rarebear et al:
I stumbled on a video which explores this topic in some depth. I’m reasonably sure that this will meet with scorn and derision among many here, but I think it makes some very valid points. It seems to me that arrogance, and narrow-mindedness, has become integral in the production of these “scientific” thought processes, and I am deeply concerned about their effects on mankind if the more enlightened citizens of this world can not moderate them.

I don’t for a second believe that humans are simply physical devices, and I find some of the statements made by Hawkings, and others quoted in the video, to be completely absurd (and very dangerous). I’d have more respect for these individuals if there was more acknowledgment that they are not infallible, and that this exploration of truth is, as it will always be, a work in progress.

ETpro's avatar

@InTheZone Very interesting link. THanks for posting it.

ETpro's avatar

Here’s another interesting article about mind versus brain. And here’s the original article by Marc Hauser..

RealEyesRealizeRealLies's avatar

@InTheZone While I appreciate your link, and found the video interesting, the author himself seemed (to me) to have that tone of “arrogance” you spoke of earlier in reference to science.

As well, in these discussions, I’m put off a bit with use of the phrase “the more enlightened citizens of this world”. It seems to draw battle lines, turning a discussion into a debate. It can easily lead to a diversion of “who’s to say who’s more enlightened than another person”?

We can’t help but editorialize our positions within the roots of our comments. Of this, I’m guiltier than most. That’s why I’m always checking myself to make sure I’m not unintentionally trying to make someone who disagrees with me feel like they are inferior in some way. Hard to do, and I’m convinced it is one of the biggest hurdles preventing cooperative understanding. We get so busy defending ego, that we cannot concentrate on the subject before us.

As to the video, I’m mostly in agreement with the message, and have a great deal to add too it actually. I like what was said. I just didn’t like the way it was said. Hawkings is an iconic figurehead in many influential circles. Ripping him to shreds like that is like toppling someones god figure. Well, I do the same with Dawkins. And the effect is generally met with the same response as a highly religious person would react to scandal about Muhammad or Jesus.

We must find some middle ground, sans dogma and ego, to discuss these issues intelligently.

Rarebear's avatar

@InTheZone Thanks. I’ll watch it when I have some time.
@etpro Printed out.

Rarebear's avatar

@InTheZone Okay, I watched the video. I’m sorry, but it’s pseudoscientific metaphysical babble. I don’t even know where to begin. It is wrong in almost every aspect of what it says, and is filled with so many logical fallacies that I started a list, scribbling furiously, and finally gave up.

Rarebear's avatar

@InTheZone And I’ll just add that I feel its anti-scientific view is actually dangerous by painting modern medicine in a evil light.

WasCy's avatar

Thanks, @Rarebear. I gave up trying to think of the words to use after “pretentious” and “boring as hell”, so I didn’t even use those then. I was also taken aback by the analogy ”... as bad as [implied] conventional medicine.”

@RealEyesRealizeRealLies… “arrogance” is another good word.

Rarebear's avatar

@ETpro OK, I read the article by Hauser. It’s a good article. My problem with it is that it already presupposes the existence of a mind and it doesn’t really address my underlying question which is, is there an actually “mind” that is separate from a “brain”.

We have a guy in the ICU now with anoxic encephalopathy. His brain is damaged. Where is his mind now?

WasCy's avatar

@Rarebear there was a time when I believed in a distinction between body, mind and spirit; a time when I believed that these three things were separate – and real. And I’d really like to have a basis to maintain that belief. I really would. But I’ve… um… changed my mind to believe in only things whose “existence” can be proved. (Even that raises fundamental questions of existence and epistemology that I’m not going to address in this post. But at least a brain is something that can be touched and felt – though I hope like hell mine isn’t ever touched and felt while I have occasion to use it.)

It would be nice to believe in god or gods and spirits and “mind”, but I can’t do that any more.

Rarebear's avatar

@WasCy Yes, I was also of that belief as well, but, like you, I’ve become an empericist in my old age.

cockswain's avatar

I really consider becoming a true empiricist to be a worthy mental achievement.

Rarebear's avatar

@cockswain You’re correct. It takes mental discipline. I certainly slip frequently, but I try to stay vigilent.

WasCy's avatar

On the other hand… I also believe in “something”, call it will or intent or soul for lack of a better word that directs a lot of the higher-level functioning of my brain. I don’t believe that “chemicals” can be self-aware, and whatever “life” is, that’s also “more than the physical brain and the body” – we just don’t know what, yet. So I’m also not a pure determinist, believing that everything I do or have done or will do was preordained by the location of my set of atomic building blocks was at the Big Bang, and the spin imparted to them then.

Rarebear's avatar

@WasCy I’m not a determinist, actually, but an empericist. I believe there is way too much randomness for things to be deterministic.

You and I differ on this, and this is the crux of my question. You said ” I also believe in “something”, call it will or intent or soul for lack of a better word that directs a lot of the higher-level functioning of my brain.” I do not. I think that my will, intent, and direction is purely an advanced biochemical and bioelectrical mechanism.

WasCy's avatar

@Rarebear

You might be right. Life may be nothing more than “a complex chemical reaction that we haven’t quite mastered yet”. But with as much as we know about chemistry now, I think if that were the case we’d be a lot closer to being able to say with certainty ”This is what life is… and make it ourselves.” I don’t know anyone ready to make that pronouncement yet.

ETpro's avatar

@Rarebear I felt the same problems with the Hauser article, but just threw it into the mix to keep the discussion going.

Rarebear's avatar

@WasCy I have no idea whether I’m right or not. But as an empericist and a skeptic, I am obligated to take the null hypothesis (in this case being that there is no mind, or soul, or whatever) and wait for evidence to show that there is.

WasCy's avatar

I understand what you’re saying, guys, and very clearly. However, the other side of the null hypothesis is that despite the mind-boggling array of goods and products and mechanisms we can make (better living through chemistry) – including replacement body parts – we haven’t managed to make a chemical / physical reaction that can animate any of those parts, create “artificial life” to re-animate a corpse which may only have needed “one more breath” or “one more heartbeat” to come back to life on its own. We can’t manufacture life, no matter how much we know about the arts and science of medicine and chemistry and manufacturing. No amount of processing power in a manufactured computer can make it “conscious” yet.

So my null hypothesis is “there’s something else” besides chemistry. But I’m also waiting for evidence of what it is; I take it as a given that there “is” something.

Rarebear's avatar

@WasCy I respect and appreciate your answer, and know that I have the greatest respect for you when I say this: It’s not a null hypothesis. You don’t have a null hypothesis that something exists without evidence. That’s a hypothesis, certainly, but it’s not a null hypothesis. A null hypothesis is always it doesn’t exist until proven.

WasCy's avatar

I appreciate the tone of the debate, @Rarebear. I think the evidence for “something” is consciousness / life itself. If it was “just chemistry”, then we can determine to incredibly close tolerances what is in the body. We can manipulate parts of the body and manufacture some pretty good approximations and/or replacement parts.

But no matter how much we know about biology, chemistry, physics, electricity and “body processes”, we can’t re-animate a corpse.

The evidence that “something did exist” occurs when you pronounce a time of death. The body in front of you is exactly what you had a minute before death… but the life force is gone. Was it simply “a chemical” that has now gone missing? That would be pretty easy to discover and replace, if so.

cockswain's avatar

@WasCy I get what you’re saying. The way I tend to look at consciousness (these days) is a pattern of brain activity and the chemistry/physics behind it. Check out that connectome TED talk I linked above. So further along the lines of what you’re saying, I like to compare our brains and what we call consciousness to that of increasingly simpler organisms. Heading back through the mammalian kingdom, back through insects, fish, and microorganisms. Where does consciousness begin? Are ants and bacteria conscious? Obviously a duck is. But if a single-cell bacteria is life, is a virus? So then you get back to the concept of a primordial stew of amino acids that somehow became RNA that “learned” (in my opinion through unknown and amazing chemistry) to become self-replicating. Heading back the other way, it eventually produced reptiles, birds, mammals, and us.

I hope I’m not coming off as condescending, I know you also know this stuff. I’m just trying to express my point.

So the increasing brain size resulted in increasing consciousness or mind or whatever you want to call “it.” The ability to reason, analyze, express, communicate are natural consequences of this evolution.

That’s the way I look at it anyways. I’m pretty excited to see where nano and biotech take us in terms of improving our minds/brains. Some futurists predict we’ll all be connected to the internet via thought one day. Our memories can be as sharp as hard drives. I’m getting a little sci-fi and tangential now, but I think it speaks about who and what “we” and our brains really are.

WasCy's avatar

@Rarebear

Consider it this way. When Priestly and Lavoisier and others were fumbling with the science that would ultimately discover “oxygen”, they weren’t specifically trying to find “that”. All they were doing, as I understand it, was attempting to prove that the physical universe was made up of more than “the four elements” (earth, air, fire and water). In fact, for most of his life Priestly believed that “air” was a “dephlogisticated” gas – “air with the fire removed” – but he had started to discover the chemical element that we breathe: oxygen.

In other words, he didn’t prove: “here is oxygen”, because he didn’t even have a way to articulate that. Maybe you’re right, and the answer is an even more fundamental particle of some sort, even one in the oxygen molecule itself: a “life and consciousness particle” that we just haven’t discovered yet. I doubt that, but I recognize the possibility.

I recognize that the brain is the electrical computing engine that determines how we sense the world and how we direct our nerves and muscles to reach for more or to withdraw. But if you believe that a thought, an idea, a judgement, a sense of wonder, love itself is “nothing more than a chemical reaction in the brain” (and I agree that the sensations in the body are certainly caused by chemical interactions with other body parts, directed and controlled by the brain) then I’ll buy you all the chemicals you need. You go ahead and assemble them to produce love. In fact, if we get a willing donor, we can even start with a fresh corpse, so that you don’t have to produce everything from scratch.

When I say that “I saw a ghost!” (or god, or an invisible pink unicorn) then I know that it’s up to me to provide some evidence of the sighting if I want the claim to be taken seriously. But no one ever asks, “Who are you to make that claim?” Why not? Is it because “a person” is self-evident? Why doesn’t a skeptic ask me to “prove myself” first, even ahead of my claim?

I could just as easily, I suppose, make the claim for being god and the center of the universe, myself, and believe that each other “person” interacting with me is just a figment of my own madness. That you only exist as an ephemera and defect in my own consciousness. For all I know, that’s the truth about life. But I don’t want to believe that, because being the only god in the universe would seem to be such a sterile and un-fun game.

Priestly never knew he had discovered “oxygen”. All he knew was that he had proven that the world was made up of more “elements” than had previously been known. I think that “consciousness itself” proves that there’s more to life than chemistry and physics. We just don’t know what that is yet.

WasCy's avatar

@cockswain

No, you didn’t come off as condescending in the least. It was, in fact, helpful to see how your thought process worked on the development of consciousness.

But the same challenge applies to you as to @Rarebear. Take a dead duck and make it live. You can have any chemicals or energy source you need, and all the time in the world. For that matter, let’s determine what “dead” means to a bacterium or virus, and you can re-animate either of those, instead.

I understand “null hypothesis”: the proposal that ‘no significant difference’ exists between two samples. But between a living person and a dead one, “life” is a pretty significant difference. Don’t know what it is, yet, but we can measure all kinds of ways to see when it has left the building.

Edit: And so I don’t miss the other part of your argument, I have no doubt that increasing our understanding of the chemistry and physics that we do have will also increase our ability to manipulate and improve the processes themselves – just like baseball players today are probably, on average, far superior to those of 100 years ago. But show me a computer that can invent “baseball”, and I’ll eat my hat.

Rarebear's avatar

Certainly (Dr. Frankenstein aside) we can’t reanimate a corpse. Nor can we create a complex organism. But 100 years ago we couldn’t fly either.

Just because something is not understood or not explained does not mean that there is not an explanation. The idea of a null hypothesis is to not accept anything on faith; to say something does not exist until it is proven to exist.

WasCy's avatar

Hmm. Well, even with Dr. Frankenstein, we still can’t reanimate a corpse. You can pull some strings and possibly stimulate some nerve cells to simulate some of the actions of a living being, but you can’t create “consciousness”. All you can do before tissue decay sets in is to make a ghastly semblance of a puppet.

Or maybe consciousness doesn’t go away, but just buries deeper. When my computer seems to “freeze”, it doesn’t really do that, it (usually) just gets stuck in some kind of endless process loop that requires all of the machine’s power to uselessly attempt to complete. It’s only a “freeze” to me because I don’t have a better way of visualizing or knowing exactly what is happening. All I see is a dead computer with the lights on. Maybe human consciousness goes into some analogous processing loop that only hides its appearance from other observers (the way people used to think that deeply unconscious or certain vegetative cases were “dead”, only deeper, to a level we haven’t yet discovered).

I’d say in counterpoint to your final paragraph that “just because a thing appears to be” a slab of animated meat does not mean “that’s all that is there”. I think we have to recognize that “of course there’s more than meets the eye”. The trick is to not presuppose what it is, I think. I won’t go writing any holy books based on my supposition that “there’s more to a body than the body” (and I will keep kicking at the holy books of those who attempt to write their own – or force others’ books on me) without proof of their own.

But there’s “something”; I just don’t know what it is. It would be foolish to say that there’s no difference between the living and the dead (the null hypothesis, after all) when it’s clear that there’s a fundamental difference – a huge difference; all the difference in the world: the difference between life and death. Living beings are alive. They have life, whatever that is.

Rarebear's avatar

@WasCy Sure they have life. They have active cellular processes with ATP production and cellular replication, etc.

I’m not being sarcastic or glib. What is “alive” is an organized organic structure with cellular process that have evolved through thousands of millions of years to work together to form an organized whole.

WasCy's avatar

I don’t take your response for sarcasm or glibness. In fact, I bow to your superior understanding of the chemical and biological processes, markers and functions. But what is the difference between your “patient” and your “late patient”? The one who died “had” life; he “was alive”. Surely he’s not just missing an essential element that escaped from his body.

I’m not being sarcastic or glib, either. There’s “something else” besides the chemical and physical processes going on. We just don’t know what that is yet.

If the argument is about “how did life begin?”, that’s another whole discussion. Which we also don’t know.

cockswain's avatar

I think, like @Rarebear states two posts above, it is a complex organized system of systems, delicately balanced, that can be collectively self-sustaining if this balance is maintained. The renal system fails, the consciousness dies as the brain cells die. Same deal with the pulmonary or cardio system. Anything that causes sustained hypoperfusion, and you’re body died; stop feeding the brain oxygen and glucose, and the self-awareness dies. We are just a massive conglomerate of different cell types, beautifully orchestrated to work together.

I think, unless it somehow spawns a new scientific discipline, this “something else” you speak of will just add to our knowledge of chemistry and/or physics. As it stands, there is a debate among physicists and chemists as to which branch of study better explains the universe. Personally I go with physics.

Regarding this reanimation concept, I think once a body is broken, you just can’t bring it back. It’s not like a heart can stop and brain cells won’t die until they are all dead. You could put a heart back in the body and it won’t start on it’s own and bring the brain back (I realize this isn’t news). This reanimation concept is more like conception. Everything starts at once, all the systems develop together in the embryo, spawning this self-sustaining, self-aware organism.

WasCy's avatar

Hmm… well, we’re getting farther afield. Since you mentioned “the universe”, I have to say that neither discipline adequately “explains” it: How did it come to “BE” in the first place?

It’s one thing to say “how” it works and how it keeps doing what it does.

Why is it – or any of us, including those damn ducks – even here? Explain that.

Rarebear's avatar

@WasCy We’re not here for any reason. The Earth had the correct conditions for abiogenesis (down @RealEyesRealizeRealLies)! and so life was formed. Evolution took care of the rest.

WasCy's avatar

@cockswain

Oliver Wendell Holmes beat you to your argument.

@Rarebear

You missed my point by about 15 billion years: Why is ANYTHING here? Physics and chemistry don’t explain – at all, not for an instant – how everything came to exist in the first place.

Rarebear's avatar

There doesn’t have to be a reason. If you believe the multiverse theory (which I don’t for the same reasons why I don’t believe in a mind or a soul—no evidence yet) which hypothesizes that there are an infinite number and type of universes out there, then statistically our particular universe is not only probable but certain to exist.

Let’s take another far more mundane example. Let’s say I were to toss a coin 10 times in a row, what are the chances that I would hit heads on every toss? Well, it’s 2 to the 10th power. But let’s say I tossed the same coin over and over again for 9 hours. What are the chances that SOMEWHERE in there you will get 10 heads in a row? Virtually certain. http://www.youtube.com/watch?v=X1uJD1O3L08

cockswain's avatar

@WasCy I like your interpretation of that poem in this context. I’m not going to offer any explanations why the universe exists. Thankfully my ego doesn’t extend that far. But I like the coin flipping analogy. There were billions of years for the coin to hit heads 10 times in a row and create amino acids and peptide bonds.

I’ve read a couple books that get into String Theory and M-Theory (“Euclid’s Window” by Mlodinow, and “The Great Beyond” by Halpern, and part of one by Brian Greene) and watched some lectures by Edward Witten and Lisa Randall. I know enough to know that if the Large Hadron Collider discovers a certain particle (the Higgs boson) it will mathematically provide very strong validation for one of those insanely complicated theories.

I got straight As in calculus and know enough to get a deep respect for how difficult these mathematical theories are to conceptualize, let alone build on and discuss with like-minded geniuses. These geniuses are a very rare breed amongst humans. While they can lead this field, 99% of humans (a number I pulled out of my ass) would struggle to explain how the formula for the volume of a sphere is derived. If understanding the formula for a sphere is pretty involved, how the hell can the human mind fathom the mathematics behind the universe/multiverse?

I know I’m getting out there now, but I watched an interesting documentary about Ray Kurzweil recently, discussing the future of nano and biotech, subjects I find fascinating. I also read excerpts from his books “The Age of Spiritual Machines” and “Fantastic Voyage.” Theoretically we should be able to create computers as complex as our brains one day. One day, create machines with even greater capability than our brains. Those machines have a better chance of answering these questions.

Plus they will invent things that will make you eat your hat.

WasCy's avatar

@Rarebear I completely agree that there doesn’t have to be “a reason” (as in “purpose”) for the universe. But it had to come into being from something, right? Tautologically speaking, there had to be a universe in order to have a universe.

That is, the energy and/or the matter to make the thing had to be “created” at some point. Physics and chemistry can’t explain “how it all came to be here”, and I think it’s impossible for them to ever do that. It’s also unlikely for priests to do that, but in a world of billions of individual explanations for “how we came to be”, one of them may be coincidentally correct in every detail. It might be equally impossible for us to say “this is how life is created; this is what life is”. But because we can’t explain what makes us “alive”, it’s equally absurd to say “it’s just this collection of chemicals” as it is to say that “Snake Mother made us” or “faeries created the First Man and First Woman in a tree” or “God did it” and leave it at that.

We can continue to explore the nano side of life – and I hope that we do – but I don’t suppose that we’re going to find a magic animating particle. I’ll be happy to read about its discovery, if so.

I’m fine with the statistics that you use to posit (apparently) how the spark occurred to start the generation of life. I have no major quibbles with evolutionary theory. I’m certainly not a religious fanatic of any stripe who says “my creation myth rules” and “I know what life is because I read a holy book of my father’s choosing”. I’m not even saying “I have a soul” or “I am a soul” and I already know I don’t got soul, so don’t go there. But what I am saying is that my consciousness, and yours and everyone else’s even that duck is not “just a collection of chemicals”.

I agree that that’s all we can see right now, but that’s not all there is. From that belief that “there must be more” it does not logically follow that I believe in every phantasm, ghost and superstition that exists in the world. But because I know that “there must be more” (because I recognize a fundamental difference between a corpse and a live person, even if I can’t say with precision what that difference is, other than some body processes) I guess I’m open to the possibility.

If someone wants to say “Man is a spirit”, I’d still join you in asking them to prove the assertion. All I’m saying is that we’re not reduced – can’t be reduced – to “just a collection of chemicals”. If we believed that on any level, then drowning a kitten – or a baby, for that matter – would be no more a criminal act than snuffing a candle.

WasCy's avatar

@cockswain I look forward to the day when I might be required to eat my hat.

cockswain's avatar

I take exception to comparing the “just a bunch of chemicals” idea as having equal validity to the Snake Mother or faeries or God ideas. I don’t like use of the word “chemicals” either, for semantic reasons. I prefer to think of it as atoms and molecules (only because I’m not well versed in anything smaller than electrons and protons). We understand how electrons move to form bonds and form molecules. We understand and can synthesize these bonds between molecules to form many different shades and variations of biomolecules. We can synthesize chemical RNA, DNA, and protein strands. We understand many, many things about cellular biology on a molecular level. These are reproducible, observable effects. So to tie our bodies and hence our brains into molecular interactions is actually a reasonable assertion. Linking such tried and tested hard science to myth is not reasonable.

Possibly advances in nanotech could cure most diseases. Stem cells could grow you a new whatever organ for transplant. Kurzweil writes of these technological bridges that could possibly lead to human lifespans reaching hundreds or thousands of years. Eat your hat Wheaties and hope for the technology to get there for us.

Just as an aside, this http://en.wikipedia.org/wiki/Euler's_formula is the most fascinating thing I know of. If this doesn’t show there is some deep order to the universe, I don’t know what does.

Rarebear's avatar

@WasCy That’s fine. My point with your point is the statement, “From that belief that “there must be more” it does not logically follow that I believe…” And the core word is “believe”.

A skeptic’s major struggle is not to “believe” anything without evidence. I have no problem with people having a core belief in something (be it a mind, soul, God, the Singularity, the Multiverse or whatever). But it’s not science. Okay, Multiverse Theory has a bunch of math behind it but there’s no experimental evidence as yet

WasCy's avatar

@Rarebear I don’t think we’re very far from agreement, anyway. I agree with your statement about a skeptic’s major struggle to avoid unfounded “belief”. I get it. I live it. But I think the other major struggle for us as skeptics is to know the limits of our knowledge. And that’s where I think we have our disagreement. I don’t think we know enough about “life” or “consciousness” to make claims about where it comes from, what it really is, or how it comes about.

You believe that “your brain is your mind”, and vice versa. I don’t think so. Obviously we know more about brains every day. We know more and more about more and more functions and processes that make our bodies and brains work, and how they work. I don’t doubt that you know far more than I do or ever will about those processes and functions and their various pathologies. But I think you’re actually assuming more knowledge than you actually have. You’re assuming (I think; you’re the only one who can correct me on this) that because you know so much about the processes and the minutia of the functions and structures… that you understand more than you actually know about the animating force itself.

I doubt if we ever will, or can; I just think that’s the “more” that we haven’t yet discovered. And if we’re lucky, we probably never will. I’m thinking Monkey’s Paw, Frankenstein and zombies that we’d be building if we got close enough to understand “life” and how to start it. It’s bad enough that we know as much as we do about stopping it. What a horror if we could create it at will.

I don’t think that it’s particularly unscientific to believe that “there is something there that we don’t know yet”. In fact, that’s one of the primary drivers of science, isn’t it? It would certainly be presumptuous and unscientific to pretend knowledge of a thing beyond testable and falsifiable hypotheses, wouldn’t it? Like saying that “consciousness is strictly chemical and physical”, for example.

We’ve known about gravity, for example, since we lived in trees, even if we didn’t even have a word for it for millennia. And we applaud the inquisitiveness that led Galileo to measure the speed of falling bodies and rate of change over distance and time, and led Newton to develop his knowledge of what gravity does and how its effects can be measured. But even now I don’t think we understand the “why” of gravity. Why is it (this is just a rhetorical question, certainly not meant to derail this excellent thread) that bodies of matter have a gravitational force? We know that they have it, but we don’t know what causes it. It’s another unknown property of matter, even after all this time spent studying matter. I doubt that it’s the “soul” of matter just as I doubt that I have a “soul” wherein resides my consciousness. But here is that same old same old gravity still working on me… and my same old same old “awareness” of it. I doubt that my awareness or the gravity itself are “just material”. I’m sure that “gravity” is a function or property of material that simply hasn’t been fully explained yet. And maybe maybe so is “consciousness”; but I doubt that very much.

RealEyesRealizeRealLies's avatar

Paradox of the Empirical Materialist… yet another.

Uses Reason, in this Age of Reason to determine that “seeing is believing” is the most accurate and proficient method of observing reality. Yet the Reason used to make that determination has never been seen.

As well, the Ancient Cosmos (purported to be without Reasoning) has somehow manifested Reasoning via humans on this thread. Yet no mechanism has ever been demonstrated to manifest Reason from that which has no Reason.
__________

The Empirical Materialist must be Reasonable enough to understand that when pursuing a “seeing is believing” methodology or world view, that there are many different forms of sight. Beyond the physical eyes, one must be capable of also envisioning. It is perfectly natural for humans to envision. We do it all the time with every object we create. The first step of which is to transduce a Thought/Spirit of Mind/Soul into an electrical impulse for the brain to initiate hand movements which author the first physical representation of the Thought/Spirit. Wess Warren has discovered that ncRNA have a direct relationship to the electrical impulses which fire the brain to move the body. It seems we are discovering that our genome is the ghost running a genetic machine which switches the brain like a traffic light complete with 24/7 recording.

Keep in mind, that the genome is an immaterial phenomenon. We cannot touch or hold the genome. The genome is the message – the Thought/Spirit. The genetic code allows us to see the Thought/Spirit.

Nothing new about that either. That’s what code does. It allows us to see the thoughts of others. Our physical eyes see the code. Our minds eye sees the thought it represents.

To one who denies the possibility for a mind to even exist, that also denies the very source of the denial.

cockswain's avatar

@RealEyesRealizeRealLies @WasCy I personally (and I think @Rarebear may share my sentiment) do not think it is outside of the realm of possibility for “mind” or “consciousness” to be forces of nature that our brains are able to tap into. I think that’s somewhat what you guys are saying. I don’t think we know everything, and do not deny any possibilities. I assess likelihoods based on my knowledge. Physics implies (but doesn’t prove) many dimensions beyond the 3 (or 4 if you include time) that we’re generally accustomed to. All of this, it seems, to find a way to unify gravity with the other known major forces of the universe (this is probably overly simplistic). So how our brains, bodies, and all associated quarks, gluons, leptons, bosons, dark matter, and other weird things I don’t understand interact with these other possible dimensions and affect our perceived reasoning is nothing I will say we have solved and can define. I think it will fall under the realm of physics to explain though. No one is saying we’ve got it all figured out. BUT, all science related to the very small is far more substantiated than any other guess. Therefore, despite not necessarily being proven true, it seems the most likely and therefore currently best explanation yet. So I think this way, but leave the possibility for all of it to be completely found wrong with a new discovery one day. And I’m totally fine and accepting of that.

WasCy's avatar

@cockswain I doubt very much whether @Rarebear will go along with your unknown / unseen “forces of nature” until he can get a meter on them in some way. I think the way that I do because I don’t subscribe to the various belief systems that endow inanimate objects with “soul” or “consciousness” of their own. So I don’t think that consciousness is any function of material, and therefore it’s a separate “thing” yet to be “discovered”, measured and otherwise quantified and/or qualified.

I just don’t know what it “is” to point to and say “I know about / I believe in that.” All I know is that there is a “something” beyond the bones and the meat (and the strings, etc.) that make me up. My brain =/= consciousness; it’s just a place where that resides.

It would be comforting if I could believe that this “consciousness” or “life” will live on somewhere after this body drops, but that’s a stretch that I can’t make. I don’t even think that it’s necessarily any kind of kinetic energy so that I would even “live on” in some form of energy later on. I expect that this consciousness is “tied to” the body (or maybe the Scientologists are right after all, and it’s separate, but only sort of “hammered in” right now).

I don’t agree with @RealEyesRealizeRealLies that our thoughts “make” us; an inarticulate person who hadn’t a rational thought in the world could still be “conscious” and “alive”. However, I think there’s an avenue of thought worth exploring there:

If all empiricists can believe is “what is clearly seen, touched, heard, measured – physical things, in other words”, then what are “thoughts”, “ideas”, “art”, “harmony” and other aesthetics? Are these purely material? I can hardly credit that. How does “material” manage to overcome entropy?

RealEyesRealizeRealLies's avatar

@cockswain “I think it will fall under the realm of physics to explain though”

The Laws of Physics do not account for the Laws of Information. Physics, in fact, is attempting to redefine the word Information in order to make it fit nicely with their model. No other discipline redefines the word. Physics is alone in its attempt to reject the basic principles of Information Theory, and that’s just too bad in our current Age of Information.

@WasCy “it’s a separate “thing” yet to be “discovered”, measured and otherwise quantified and/or qualified.”

That’s what language does. That’s why there are seventy language tests used in Cognitive Studies to determine a patients level of Consciousness.

Language is very quantifiable. We’re doing it right now. Information Theory is the math which determines how much redundancy, noise reduction, and error correction must be applied to ensure any given communication between sender A to receiver B. That’s why we can map different formulas to our thoughts. Some use the formula of English. Some use Chinese. But the meaning is the same.

Multiple separate mediums can be used to represent the exact same meaning.

“The sky is black” means the same thing no matter what language you communicate it with.

Meaning is separate from Matter, and that’s why Empiricists claim that Meaning doesn’t Matter. By definition, the Empiricist only sees Matter. They cannot allow themselves to see meaning because that would reject their Empiricism. It’s not Matter.

It’s all linguistics. And the Empiricist should be satisfied with these words before us as a physical representation of non physical meaning. Even the language of Mathematics. A physical representation of a non physical platonic form. So too is the relationship between Mind and Brain.

RealEyesRealizeRealLies's avatar

@WasCy ”...an inarticulate person who hadn’t a rational thought in the world…”

Oh right… find me one of those.

Rarebear's avatar

@WasCy I see where you’re going now. I claim no special knowledge of how the brain works vis a vis the mind. My point is that all I know of is a brain. When someone suffers an anoxic brain injury their personality is gone, and their higher neurologic functioning is damaged or gone. When the organic structure is damaged, the biochemical/electrical processes are damaged. Kaput.

Consciousness is turned on when the reticular activating system is turned on, and both hemispheres are functioning. If any of those are damaged or chemically altered, you are unconscious (or asleep or drugged).

RealEyesRealizeRealLies's avatar

@WasCy “How does “material” manage to overcome entropy?”

Material IS entropy.

The best depiction of the universe is the one played as static on an untuned radio or TV. That “noise” IS the universe. That’s all there is in our entire realm of existence, aside from Mind.

Science shows us images of a nebula using extended spectrum imaging. Though it seems still, and apparently a real physical object, it is not. It does not exist until humans define it into existence. That nebula, or mountain, or mudslide, is constantly changing, as a convoluting ocean of static in an endless morphing state. The nebula is swirling just like smoke from a cigarette.

We humans are on a timeline so slow it tricks us into defining these apparent objects as physical reality. But we’ve only defined a temporary observable state, which is changing, even as we observe it.

Imagine slowing down the rapid static snow on a TV to the point where the black and white dots could actually be seen jumping from one state to the other. At times, we would observe an over accumulation of black dots at a particular coordinate. We observe and name it. We call it “nebula”, or “mountain”, and thereby define a physical object into existence. Humans harness entropic noise (the universe), and manipulate it into a physical representation of our thoughts about it. The object doesn’t exist until a mind defines it into existence.

Imagine speeding up the swirling formlessness of the universe to the point where star clusters and solar systems arose within seconds. 14 billion years of constant change depicted in a single moment. It would look like static on a television set. There would be no method for defining any particular observation because it convulses so rapidly. It is sense-less, until a Mind applies sense upon it.

Information Theory goes a long way towards helping us understand what entropy actually is. Noise reduction is of major concern when communicating any Thought/Spirit from one Mind to another. There are mechanisms (tools) which allow us to overcome the effects of entropic decay upon any transmitted signal. Error Correction, Redundancy, Signal Boosting… all tools for overcoming the unwanted effects of Energy and Matter upon our communications.

The volume switch on a radio is an obvious tool which allows signal boosting to overcome the effects of Material Atmospheric Obstacles. The only “thing” preventing a clear signal is the effects of unharnessed material objects. The Storm Cloud, the Rusting Antenna, The Solar Flare… all material objects which apply entropic decay upon our communications. The material realm is literally preventing the union of our Mind/Souls with any given Thought/Spirit.

As we communicate on this thread, we harness photons, electricity, matter, and repurpose them as symbolic alphabetic tools for expressing our Thought/Spirit to one another. Those apparent physical objects are no longer random. They have been redirected, repurposed, to perform the specific tasks of our Will.

The bricks in a building are repurposed matter obeying the direction of the code we apply upon it to represent a specific mindful purpose of a school or hospital. Those structures would never arise unless a mind harnessed entropy, and formed it into a symbolic representation of an original Thought/Spirit of a particular Mind/Soul. Everything else is the entropic decay of noise produced by the chaos of energy and matter.

Rarebear's avatar

“Though it seems still, and apparently a real physical object, it is not. It does not exist until humans define it into existence.”

I couldn’t disagree with this more. Philip K. Dick said it best: “Reality is that which, when you stop believing in it, doesn’t go away.”

The nebula is there whether we see it and define it or not.

RealEyesRealizeRealLies's avatar

“The nebula is there whether we see it and define it or not.”

The word “nebula” is only a title humans attribute to a particular state of atomic particles. They are constantly changing. What we call The Crab Nebula is nothing more than a source of entropic decay at a particular space/time coordinate. It will vanish like a cloud in the desert. Shall we name every cloud we see? Or just the ones that consist of …an interstellar cloud of dust, hydrogen gas, helium gas and other ionized gases.? One day, it may look like The Horse Head Nebula. Where will the Crab Nebula be then?

Philip K. Dick should consider the possibility that “Reality is that which, when you start believing in it, appears as defined by the observer”.

Upon observing the first Quasar, the only name we have for it is IT or THA?. It’s not a Quasar until we call it one, and thereby define what the word “Quasar” means. Naming is one thing, but meaning is created upon definition at the sentence level. The current state of molecules is now meaningful because we attributed meaning to them. Before that, they were just random noise constantly convulsing under the pressures of chaos.

Even the word “Chaos” is a descriptive term which attributes meaning to meaninglessness. Certainly “Chaos” is meaningless until a mind attributes meaningful qualities and characteristics upon it. Certainly nothing meaningful could arise from meaninglessness.

CWOTUS's avatar

@Rarebear

You said, “When someone suffers an anoxic brain injury their personality is gone, and their higher neurologic functioning is damaged or gone. When the organic structure is damaged, the biochemical/electrical processes are damaged. Kaput.”

What if it’s not “gone” or “kaput”? That is, the way we interact with people and personalities is through our bodies: in addition to the obvious spoken word (or signing or writing) we smile or frown, adjust our bodies in certain ways, raise our eyebrows and all sorts of non-verbal ways. This is how we manifest our personalities. If the tragedy happens that our brains (which we realize are the control centers for the body) are profoundly damaged, then we see no evidence for personality and assume that it’s “gone”. If you knocked on my door and I didn’t answer, then you might assume that I was “not home”. But what if I had suffered a crippling injury – and maintained full consciousness – and heard you knocking, knew who you were and wanted to let you in… but I couldn’t move a muscle to say “I’m here; come in! Please knock down the door and come in!” All you see is the blank house front, the unanswered door (maybe day after day) and think: “that personality is gone”.

You may be right. The mind may be something so ephemeral and otherwise incapable of interacting with the physical universe outside of its control of its own body via its brain, that we assume that “his brain is gone, and therefore his personality is, too”. (I’m thinking of Patrick Swayze in Ghost, and the horror of consciousness unable to interact with loved ones. The movie was too pat, I think.)

I believe that I would rather think that the mind is turned off when the body and brain are, because the idea of free-floating and “physically incapable” (100% completely incommunicado) consciousness is abhorrent to me. But I still think that they’re separate things, ultimately.

CWOTUS's avatar

@RealEyesRealizeRealLies

The point I was trying to make with @Rarebear and material / entropy is that if consciousness is strictly a function of the multiplicity of neurons in our complex brains, then that doesn’t explain how the “material” brain can overcome the entropy of the universe.

I guess we could agree (we pretty much have to) that “life” overcomes entropy. That is, living things make more and more complex arrangements of their environments and the special materials / minerals & chemicals therein to make their own bodies and from there their own homes and micro-environments. I agree with you, in essence, that “material” could not do that. That’s why I’m arguing that “life” and “consciousness” are non material – even though I make no claim as to what they “are”. (I also don’t think that life and consciousness are a form of kinetic energy; they’re really not something of the physical universe that we can study under microscopes, I think.)

Rarebear's avatar

LIfe does not overcome entropy. Your brain is locally organized, sure, but in the act of becoming organized much more energy was produced. You can’t just look at a local system—you need to look at the whole system.

It’s off topic, but Creationists use the same argument to try to disprove evolution. It’s fallacious because much more energy was used (and wasted) in producing organisms than are evident in the organisms themselves.

@RealEyesRealizeRealLies Sure, we call it a “nebula”. But even if we didn’t call it anything, it would still exist. The label changes nothing.

RealEyesRealizeRealLies's avatar

”...even if we didn’t call it anything, it would still exist…”

What would exist? “It”? At first observation, the Nebula doesn’t exist at all. Only an “IT” or “THA?” exists. We describe IT as a cloudy mass of randomly moving particles under the pressures of chaos. Then we name IT’s current configuration with an identifier of Nebula. IT’s current configuration changes fast. So fast that IT is not exactly the same as IT was just a second ago. One day soon, IT won’t be qualified to be called a Nebula any longer. The same principle holds true for a smoke ring from a cigarette. IT’s only a smoke ring for a short moment, and then IT becomes something entirely different very quickly. And IT was only a smoke ring because that’s what we named IT.

If no one ever observed IT, then the consciously identified platonic form of Nebula would never have existed. The name we assign a particular observation is simply a motif. There is no assurance that motif will catch on or be accepted universally. What you call a Nebula, someone else calls The Eye of Zeus. From the point of being named, neither one remains the same. Falling to the pressures of chaos, every observation is under constant decay.

Nothing physical in this universe is the same right now as it was just a single millisecond ago.

“The label changes nothing…”

The more precise the labels we apply to physical observation, the more we understand that the entire physical realm is changing constantly. Labels allow us to designate more precise measuring units, to determine that reality right now is different than reality just a moment ago.

Whether we use English or Math, precise identifiers (labels) are exactly what allows science to author accurate descriptions.

cockswain's avatar

@RealEyesRealizeRealLies I don’t get your point. Yes, all particles in the universe are moving and everything is in a different position than a moment ago. Are you saying we should give things better names to reflect this changing state? What would be the benefit of that for practical purposes? Should I call the shed in my yard a shed, or something that better describes it’s transient nature?

RealEyesRealizeRealLies's avatar

“Are you saying we should give things better names to reflect this changing state?”

That’s what Science is purported to pursue. The more detailed our descriptions of our observations, the more trustworthy our predictions for the next hypothesis.

“Should I call the shed in my yard a shed, or something that better describes it’s transient nature?””

You may describe “IT” in whatever manner you please. You can use smoke signals, color codes, sound waves, or scribbles in the sand. But however you describe “IT”, your descriptive term only applies to the very moment you describe “IT”, and not a second before, and not a second after.

Your description of “IT” is incomplete upon the moment it manifests.

CWOTUS's avatar

@Rarebear

You said “life does not overcome entropy”, but I think you’re… ahem… dead wrong on that.

It seems to me that life is the only thing that does overcome entropy, even if each individual life only does that temporarily. It’s “life”, after all, that accretes the raw materials of Earth into organized form (the bodies of all living things, and eventually their homes as well) for use in pursuing its main agenda: producing more copies of itself. Over the billions of years that we agree the planet has been here it has gotten to the point where we look over the surface of the planet (and down in the oceans, up in the skies, and even deep in the soil itself) and can hardly find a place that hasn’t been somehow “organized” by the forces of life.

When a body gives up whatever “life force” animates it, then the processes of entropy (aided by other forms of life!) begin to take over and “disorganize” and decompose it back to a near-elemental stage for eventual use by other life forms.

cockswain's avatar

@RealEyesRealizeRealLies So how would that improve basic communication? Instead of banana I have to give it and every other noun a much longer description? Wouldn’t it take me a really long time to say, “I’m taking the truck to Lowe’s to get some plywood and nails?”

Rarebear's avatar

@CWOTUS I am not, “ahem, dead wrong”. I am correct. You don’t look at entropy in an isolated area of organization such as in a biological construct or a rock crystal. You have to look at the system as a whole. Entropy doesn’t mean that you can’t have local areas of organization.

Rarebear's avatar

@RealEyesRealizeRealLies “What would exist? “It”? At first observation, the Nebula doesn’t exist at all.”

That’s a very postmodern point of view, a philosophy which I reject outright. There is an objective reality. Something doesn’t have to be perceived to exist.

RealEyesRealizeRealLies's avatar

@cockswain “So how would that improve basic communication?”

Details are most beneficial to communication.

@cockswain “Wouldn’t it take me a really long time to say…”

All code is reducible to a factor of one bit. The process of “taking the truck to Lowe’s” is reducible to “Friday trip” and that could be reduced to ”&^” as a sticky note on the refrigerator meaning the exact same thing. To anyone who understood the symbol ”&^”, they would have as much detail as possible.

@Rarebear “There is an objective reality”.

Nothing I’ve said contradicts that. Objective reality lets us understand that our descriptions are subjective, and never a full description encompassing the entirety of the phenomenon described. The phenomenon is not the description any more than the brain is the mind. One represents the other. They are separate agents, one based in matter, the other in thought.

@Rarebear “Something doesn’t have to be perceived to exist.”

Neither must it exist to be perceived. Unicorns, faeries, quintic equations… all brought into existence without first being perceived. And these phenomenon are described by their creator in the same manner as describing a dust cloud as a Nebula.

Answer this question

Login

or

Join

to answer.

Mobile | Desktop


Send Feedback   

`